LSAT and Law School Admissions Forum

Get expert LSAT preparation and law school admissions advice from PowerScore Test Preparation.

 Administrator
PowerScore Staff
  • PowerScore Staff
  • Posts: 8916
  • Joined: Feb 02, 2011
|
#38404
Complete Question Explanation
(The complete setup for this game can be found here: lsat/viewtopic.php?t=15027)

The correct answer choice is (C)

This is a variation on the List question, requiring you to identify a possible list of the middle five variables, from second to sixth. Use the same List question technique outlined under Question #1, but with a twist: stay mindful of the two missing variables, as their placement could end up violating the rule you are testing.

Begin by testing the most “obvious” rule—P must be 4. Answer choice (E) clearly violates this rule and is therefore incorrect.

Next, test the VW Block, which is violated in answer choice (B). Additionally, notice that the variable list in answer choice (A) places V in 6. To comply with the VW Block, you need to ensure that W is 7. This leaves R to be 1, in clear violation of the Q R sequence. Answer choice (A) can therefore be eliminated as well.

Speaking of the Q R sequence, examine the remaining two answer choices:

In answer choice (C), the sequence can be satisfied by placing R in 7, leaving us with T in 1. This arrangement appears to satisfy all four rules in the game. By contrast, in answer choice (D) neither Q nor T can be last: the former would violate the Q R sequence, whereas the latter would create a ST Block. Therefore, answer choice (D) cannot be true and is incorrect. Answer choice (C) could be true and is correct.

Get the most out of your LSAT Prep Plus subscription.

Analyze and track your performance with our Testing and Analytics Package.